How to Factor 4n^3-49n

Answers

Answer 1

Answer:

n (2n-7) (2n+7)

Step-by-step explanation:

Fyi you can use the app photo math you just take a pic of the problem and it gives you the answer and explains the steps and it is free.

How To Factor 4n^3-49n

Related Questions

Which rules of exponents will be used to evallate this expression? Select three options.

Answers

I need the answer choices and the expression you’re referring to

Find the measure of the arc or angle indicated. Assume that lines which appear tangent are tangent.

Answers

Given:

In the given circle, the measure of major arc TV is 220 degrees.

To find:

The measure of angle TUV.

Solution:

In a circle, the measure central angle of arc is equal to its corresponding arc measure and according to central angle theorem, the measure of subtended angle on an arc is half of the measure of central angle of that arc.

So, the measure of subtended angle is half of the measure of its corresponding arc.

[tex]m\angle TUV=\dfrac{1}{2}\times \text{Measure of major arc TV}[/tex]

[tex]m\angle TUV=\dfrac{1}{2}\times 220^\circ[/tex]

[tex]m\angle TUV=110^\circ[/tex]

Therefore, the measure of angle TUV is 110 degrees.

Which descriptions from the list below accurately describe the relationship between ∆ABC and ∆DEF? Check all that apply.
A. Same area B. Same size C. Congruent D. None of the above

Answers

Answer:

D. None of the above

Step-by-step explanation:

The two right triangles have different sizes. Therefore, their areas cannot be the same as well.

Congruent triangles have the same three angles that are congruent to each other and three side lengths that are congruent or equal to each other. The two triangles only have equal angles bit different corresponding side lengths. Therefore, they cannot be congruent.

The correct answer is "None of the above".

Answer:

PROPORTIANAL SIDE LENGTHS

Step-by-step explanation:

I JUST TOOK THE TEST

Joe drives for 3 hours and covers 201 miles. In miles per hour, how fast was he driving?​

Answers

Answer:

67 mph

Step-by-step explanation:

201/3 = 67

 Marsha has a bag that contains 4 green marbles, 8 yellow marbles , and 20 red marbles . If she chooses one marble from the bag, what is the probability that the marble is not red?

PLEASE HELP IF YOURE GOOD AT GEOMETRY!!

Answers

Answer:

C. 3/8

HOPE THIS HELPS :)

Answer:

c. 3/8

Step-by-step explanation:

first you need the denomerator by adding all marbles together which equals 32. now for the munerator you need the sum of the green and yellow marbles. this equals 12. so your fraction is 12/32. next we simplify. we can divide both numbers by 4. getting us a fraction of 3/8.

A road has a scale of 1:50 000 The length of a road on the map is 8.5cm.Work out the length of the real road in kilometres

Answers

Answer:

ok so

8.5*150000

1275000 cm into kilometers is

12.75 kilometers

Hope This Helps!!!

Which diagram represents the hypothesis of the converse of corresponding angles theorem?

Answers

Answer:

the first diagram

Step-by-step explanation:

first one

HELP ME PLEASEEEEEEEEEEEEEE

Answers

Answer:

a

Step-by-step explanation:

Câu 1 (2 điểm). Cho hệ các vector

U = {(1,2,2); (0,-1,-1);(2,3,3);(1,0,0).

a) tìm số chiều và một cơ sở W của không gian con sinh bởi hệ vector U

b) tìm tham số k để u=(2,3,k^2 +1) là một tổ hợp tuyến tính cảu W, và suy ra [u]w

giải hộ mình với

Answers

Answer:

Step-by-step explanation:

Question 1 of 10
Simplify this algebraic expression completely.
5y-3(y + 2)

Answers

Answer:

2y -6

Step-by-step explanation:

5y-3(y + 2)

Distribute

5y -3y - 6

Combine like terms

2y -6

Complete the table of ordered pairs for the linear y=2x-8

Answers

Given:

Consider the below figure attached with this question.

The linear equation is:

[tex]y=2x-8[/tex]

To find:

The values to complete the table of ordered pairs for the given linear equation.

Solution:

We have,

[tex]y=2x-8[/tex]

Substituting [tex]x=0[/tex] in the given equation, we get

[tex]y=2(0)-8[/tex]

[tex]y=0-8[/tex]

[tex]y=-8[/tex]

So, the value for first blank is -8.

Substituting [tex]y=-2[/tex] in the given equation, we get

[tex]-2=2x-8[/tex]

[tex]-2+8=2x[/tex]

[tex]\dfrac{6}{2}=x[/tex]

[tex]3=x[/tex]

So, the value for second blank is 3.

Substituting [tex]x=2[/tex] in the given equation, we get

[tex]y=2(2)-8[/tex]

[tex]y=4-8[/tex]

[tex]y=-4[/tex]

So, the value for third blank is -4.

Therefore, the required complete table is:

     x           y

     0         -8

    3         -2

     2          -4

Plsssss ans I am suffering

Answers

I. The multiples of three are in the third and sixth column ii. The multiples of 6 are in the sixth column b. The fifth column c. Yes

Moving to another question will save this response.
1 points
Save Answer
Question 12
Mr Espent 65% of his salary on household expenses, and 15% of the remainder on travelling expenses and was finally left with R9 500. How much was his salary?​

Answers

Answer:

rs.1680.67

Step-by-step explanation:

His salary = x

remaining % = 100 - 65 = 35%

= 100 - 15 = 85%

x × 35/100 × 85/100 = 500

x = 1680.67

Six consecutive numbers add up to a total of 69.what is the highest of these numbers?

Answers

Answer:

14

Step-by-step explanation:

9+10+11+12+13+14=69

The [tex]HIGHEST[/tex] of these numbers is [tex]14[/tex]

Let the [tex]CONSECUTIVE[/tex] numbers be : [tex]a, a+1, a +2,a+3,a+4,a+5[/tex]

Taking the sum :

[tex]a + a + 1 + a + 2 + a + 3 + a + 4 + a + 5 = 69\\6a + 15 = 69\\6a = 69 - 15\\6a = 54\\a = 54 / 6\\a = 9[/tex]

[tex]HIGHEST[/tex] value = [tex]a + 5 = 9 + 5 = 14[/tex]

Hence, [tex]HIGHEST[/tex] value = [tex]14[/tex]

Learn more : https://brainly.com/question/15974141

A group of 3 boys is sharing 4 small pans of rice krispie treats. What is the total amount of rice krispie treats each boy will get?

Answers

Answer:

each boy will get 1.33333333333 of a pan

Step-by-step explanation:

Answer:

Each boy in that group would get about 1 and 1 third of a pan of treats or 1.33 repeating

Step-by-step explanation:

To find this you would take the numbers of pans (4) and divide it by the number of boys (3).

4 / 3 = 1.33333

0.7(1.5 + y) = 3.5y - 1.47

Answers

Answer:

y = 0.9

Step-by-step explanation:

1.05 + 0.7y = 3.5y - 1.47

-3.5y + 0.7y = -1.47 - 1.05

-2.8y = -2.52

y = 9/10 = 0.9

Answer:

[tex]\textbf{HELLO!!}[/tex]

[tex]0.7\left(1.5+y\right)=3.5y-1.47[/tex]

[tex]1.05+0.7y=3.5y-1.47 \gets \textsl{Expand}[/tex]

[tex]1.05+0.7y-1.05=3.5y-1.47-1.05 \gets Subtract\; 1.05 \from\:both\:sides[/tex]

[tex]0.7y=3.5y-2.52[/tex]

[tex]0.7y-3.5y=3.5y-2.52-3.5y[/tex]

[tex]\mathrm{Subtract\:}3.5y\mathrm{\:from\:both\:sides} \nwarrow[/tex]

[tex]-2.8y=-2.52[/tex]

[tex]\frac{-2.8y}{-2.8}=\frac{-2.52}{-2.8} \hookleftarrow \mathrm{Divide\:both\:sides\:by\:}-2.8[/tex]

[tex]\boxed{\boxed{\underline{\textsf{\textbf{y=0.9}}}}}[/tex]

[tex]\bullet \bullet \bullet \bullet \bullet \bullet \bullet \bullet \bullet \bullet \bullet \bullet \bullet[/tex]

[tex]\textbf{HOPE IT HELPS}[/tex]

[tex]\textbf{HAVE A GREAT DAY!!}[/tex]

In mixture A of candy ingredients there are 3 parts of milk-chocolate for every Z parts peanut separate mixture B. there are 4 parts of nougat for every 3 parts of caramel equat parts of A and combined in one mixing bow what is the ratio of peanut butter to nougat in the bowl

Answers

3m=Z :A
4n=3c :B

If it’s a1:1 for A:B
z:4n

Feel like you left out something but this is what I got from info provided

Answer:

D. 7:10

Step-by-step explanation:

The Correct Answer

Suppose that from a group of 9 men, 1 will be randomly chosen for a dangerous assignment, and suppose that the chosen man will be killed during the assignment with a probability of 1/6. If Mark is one of the 9 men, what is the probability that he will be chosen for the assignment and killed during the assignment

Answers

Answer:

1/54

Step-by-step explanation:

1/9 x 1/6

What is the range of f(x) = 3X?
O A. All real numbers greater than 3
O B. All real numbers greater than or equal to 3
O C. All positive real numbers
O D. All real numbers

Answers

D all real numbers (-∞, ∞)

Help me with this answer I don’t it

Answers

Answer:

f(-2) = g(-2) this is the answer

HELP HELP HELP
Solve this

Answers

Answer:

What is the cos theta for, i would use sin to solve for theta and then we would get 41.25 degrees.

Step-by-step explanation:

If f(x) = x -2 and g(x) = 2x – 6, then g(4)/f(3) =​

Answers

Answer:

Step-by-step explanation:

(2×4-6)/(3-2)=2

Answer:

[tex]{ \tt{f(x) = x - 2}} \\ { \bf{f(3) = 3 - 2 = 1}} \\ \\ { \tt{g(x) = 2x - 6}} \\ { \bf{g(4) = 2(4) - 6 = 2}} \\ \\ { \boxed{ \tt{ \frac{g(4)}{f(3)} = \frac{2}{1} = 2}}}[/tex]


Ibrahim heeft een bijbaantje op de markt. Hij berekent zijn inkomsten met de formule
inkomsten in €=5+3,50 x tijd in uren. Leg de formule uit.

Answers

Answer:

Ibrahim gets 5 fixed and 3.5 per hour.

Step-by-step explanation:

Ibrahim has a side job at the market. He calculates his income with the formula income in € = 5 + 3.50 x,  time in hours. Explain the formula.

Here, the fixed income is 5.

the income per hour is 3.5.

So, Ibrahim gets 5 fixed and 3.5 per hour.

N/A questions does not exist

Answers

Answer:

ok

Step-by-step explanation:

why did you put it

Write two rational and three irrational number that are between 3and 4

Answers

a)rational number :

1)(3+4 )/2

=7/2 =3.5

2)(3.5+4)/2

=7.5/2 =3.25

b)irrational number :

1)10/3=3.33....

2)11/3=3.66....

......I hope it will help you. ..

Answer:

The real numbers, which can be represented by the ratio of two integer numbers, are called rational numbers, say P/Q where Q is not equal to zero.

The actual numbers that cannot be expressed as the two integer ratio are called irrational numbers.

Step-by-step explanation:

a)rational number :

1)

(3+4 )/2

=7/2 =3.5

2)

(3.5+4)/2

=7.5/2 =3.25

b)irrational number :

1)

10/3=3.33....

2)

11/3=3.66....

3)

√13

how would I classify a triangle which has a angle of 49 and 82, acute, right, or obtuse?

Answers

9514 1404 393

Answer:

  acute

Step-by-step explanation:

The third angle is ...

  180° -49° -82° = 49°

So, the triangle has two angles the same, 49°. When two angle are the same, the triangle is an isosceles triangle.

The largest angle, 82°, is less than 90°, so is an acute angle. The classification acute, right, or obtuse is based on the measure of the largest angle.

The triangle is an acute isosceles triangle.

What is the reciprocal of tanB in the triangle below?

Right triangle A B C is shown. C is the right angle and side A B is the hypotenuse.
tanC
tanA
tan-1C
tan-1A

Answers

Answer:

Tan A

Step-by-step explanation:

Tan B = opposite / Adjacent = AC / BC

Reciprocal of Tan B = 1 ÷ Tan B

1 ÷ Tan B = 1 ÷ AC/BC = 1 * BC / AC = BC / AC

Reciprocal of Tan B = BC / AC

the reciprocal of tan B is equivalent to :

Tan A = opposite / Adjacent = BC / AC

Hence, the reciprocal of Tan B is Tan A

Answer:

Note: Images are not in order. Check page number on pictures to make sure you have the right Answer.

Have a Good Day! God bless!

Step-by-step explanation:

Question 6 “A”

Question 7 “D”

Question 8 “B”

Question 9 “B”

Question 10 “A”

Note: Answers From 1 to 5 in order here:

Question 1 “ B”

Question 2 “A”

Question 3 “B”

Question 4 “D”

Question 5 “D”

I need help with this!

Answers

Answer:

1.  y=5   x=1

2.  y=4   x=4/5

3.  y=2   x=2/5

Step-by-step explanation:

Plug the y-values in for y:

1. 5=5x

2. 4=5x

3. 2=5x

Then solve for x:

1. x=1

2. x=4/5

3. x= 2/5

Hope this helps!

Find the value of -6 + 8(-3) ÷ 4
please help me .

Answers

Answer:

-12

Step-by-step explanation:

cause from the rule

we should always solve from bracket so -3×8= -24

and -24/4= -6

and -6-6

= -12

Find the area of the following shape:

Answers

Answer:

36cm^2

Step-by-step explanation:

total area: 6x(4+3)=42

total area excluding the space: 42-(2x3)=36

Answer:

36 cm squared

Step-by-step explanation:

To solve this problem, I first construct a line. (shown in yellow in the first photo)

I then find the area of the top rectangle. (6 cm * 4 cm = 24 cm squared.)

Next, I find the area of the lower rectangle. But...to do that I have to find the length of the line that I constructed. To do this, I do  6cm-2cm=4cm.

Then I can find the area of the lower rectangle. (4cm*3cm=12cm squared.)

add up the area of both of the rectangles and.........12+24=36 cm squared

Other Questions
I need help Ill give u brainlest What is the average score of runa 140,96 and 13? solve marked question only ! plz Hi everyone I am on my sis brainly can u help me with this Animals and plants have that allow us to sort them into groups. For example, birds and mammals both breathe using But give birth to live young, unlike which lay eggs. I need help!!! Parallel lines p and q are cut by two non parallel lines m and n as shown in the figure the value of x is degrees and the value of y is degrees Find two integers whose sum is -23 and product is 132 savaine are you serious Indias Golden Age began as a direct result ofan increase in trade.a takeover of other Asian countries.a decrease in government dependence.the fall of the Mauryan empire jeo eun achiminda. chingu . Convex angles help me There was a burglary in a nearby house and the police are taking an inventory of missing items. Listen to the audio and write down the rooms and the missing items in the exact order you hear them. D 51 F E 17 m Find the length DE. O 10.7 m O 13.2 m O 13.8 m 0 21.0 m O 21.9 m 0 27.0 m qual a reacao e o pemsmemto do jovem ao descamsar a sambra da arvore put capital fulstop n apostrope in the para graph What industry did not suffer during the Great Depression? O A. The banking industry B. The manufacturing industry O C. The movie industry D. The farming industry Stats question what are the main difference between frequenting and Bayesian 1. When the following oxidation-reduction reaction in acidic solution is balanced, what is thelowest whole-number coefficient for Rb*(aq)?Rb(s) + Sr?+(aq) Rb+ (aq) + Sr(s) Can someone answer with steps and explanation? Thanks. What is accessibility ? 4(x+9)=8x-7Solve for x